Difference between revisions of "2017 AIME I Problems/Problem 14"
R00tsofunity (talk | contribs) |
|||
(9 intermediate revisions by 2 users not shown) | |||
Line 3: | Line 3: | ||
Let <math>a > 1</math> and <math>x > 1</math> satisfy <math>\log_a(\log_a(\log_a 2) + \log_a 24 - 128) = 128</math> and <math>\log_a(\log_a x) = 256</math>. Find the remainder when <math>x</math> is divided by <math>1000</math>. | Let <math>a > 1</math> and <math>x > 1</math> satisfy <math>\log_a(\log_a(\log_a 2) + \log_a 24 - 128) = 128</math> and <math>\log_a(\log_a x) = 256</math>. Find the remainder when <math>x</math> is divided by <math>1000</math>. | ||
− | ==Solution== | + | ==Solution 1== |
The first condition implies | The first condition implies | ||
Line 37: | Line 37: | ||
<cmath>x = 2^{192}</cmath> | <cmath>x = 2^{192}</cmath> | ||
− | We only wish to find <math>x\bmod 1000</math>. To do this, we note that <math>x\equiv 0\bmod 8</math> and now, by the Chinese Remainder Theorem, wish only to find <math>x\bmod 125</math>. By Euler's Theorem: | + | We only wish to find <math>x\bmod 1000</math>. To do this, we note that <math>x\equiv 0\bmod 8</math> and now, by the Chinese Remainder Theorem, wish only to find <math>x\bmod 125</math>. By [[Euler's Totient Theorem]]: |
<cmath>2^{\phi(125)} = 2^{100} \equiv 1\bmod 125</cmath> | <cmath>2^{\phi(125)} = 2^{100} \equiv 1\bmod 125</cmath> | ||
Line 51: | Line 51: | ||
Using [[Chinese Remainder Theorem]], we get that <math>x\equiv \boxed{896}\bmod 1000</math>, finishing the solution. | Using [[Chinese Remainder Theorem]], we get that <math>x\equiv \boxed{896}\bmod 1000</math>, finishing the solution. | ||
− | == Alternate solution == | + | ==Solution 2 (Another way to find a)== |
+ | |||
+ | <cmath>\log_a(\log_a(\log_a 2) + \log_a 24 - 128) = 128</cmath> | ||
+ | |||
+ | <cmath>\implies \log_a(\log_a 2))+\log_a(24)=a^{128}+128</cmath> | ||
+ | |||
+ | <cmath>\implies \log_a(\log_a 2^{24})=a^{128}+128</cmath> | ||
+ | |||
+ | <cmath>\implies 2^{24}=a^{a^{(a^{128}+128)}}</cmath> | ||
+ | |||
+ | Obviously letting <math>a=2^y</math> will simplify a lot and to make the <math>a^{128}</math> term simpler, let <math>a=2^{\frac{y}{128}}</math>. Then, | ||
+ | |||
+ | <cmath>2^{24}=2^{\frac{y}{128} \cdot 2^{\frac{y}{128} \cdot (2^y+128)}}=2^{\frac{y}{128} \cdot 2^{y \cdot (2^{y-7}+1)}}</cmath> | ||
+ | |||
+ | <cmath>\implies 24=\frac{y}{128} \cdot 2^{y \cdot (2^{y-7}+1)}</cmath> | ||
+ | |||
+ | <cmath>\implies 3 \cdot 2^{10}=y \cdot 2^{y \cdot (2^{y-7}+1)}</cmath> | ||
+ | |||
+ | Obviously, <math>y</math> is <math>3</math> times a power of <math>2</math>. Testing, we see <math>y=6</math> satisfy the equation so <math>a=2^{\frac{3}{64}}</math>. Therefore, <math>x=2^{192} \equiv \boxed{896} \pmod{1000}</math> ~[[Ddk001]] | ||
+ | |||
+ | == Alternate solution 1 == | ||
If you've found <math>x</math> but you don't know that much number theory. | If you've found <math>x</math> but you don't know that much number theory. | ||
Line 65: | Line 85: | ||
<cmath>2^{192} \equiv 336*336 \equiv \boxed{896}\bmod 1000</cmath> | <cmath>2^{192} \equiv 336*336 \equiv \boxed{896}\bmod 1000</cmath> | ||
+ | == Alternate solution 2 == | ||
+ | |||
+ | Another way to find <math>x \bmod 1000</math> using modular arithmetic. | ||
+ | In the same way as solution <math>1</math>, we can find that. | ||
+ | <cmath>x\equiv 21\bmod 125, x\equiv 0\bmod 8.</cmath> | ||
+ | <cmath>x = 8m = 125n+21</cmath> For some positive integers <math>m</math> and <math>n</math>. | ||
+ | Taking the equation mod <math>8</math> gives <cmath>5n+5 \equiv 0\bmod 8</cmath> | ||
+ | <cmath>n \equiv 7\bmod 8</cmath> | ||
+ | <cmath>n = 8k-1</cmath> For some positive integer <math>k</math>. | ||
+ | Plug this back into the original equation. | ||
+ | <cmath>8m = 125(8k-1)+21</cmath> | ||
+ | <cmath>8m = 1000k-104</cmath> | ||
+ | <cmath>x = 8m = 1000k - 104</cmath> | ||
+ | <cmath>x \equiv -104 \equiv 896\bmod 1000</cmath> | ||
+ | <cmath>x \equiv 896\bmod 1000</cmath> | ||
+ | |||
+ | ~sdfgfjh | ||
==Video Solution by mop 2024== | ==Video Solution by mop 2024== | ||
https://youtu.be/E-7YQ9ND5Ms | https://youtu.be/E-7YQ9ND5Ms |
Latest revision as of 14:29, 8 September 2024
Contents
Problem 14
Let and satisfy and . Find the remainder when is divided by .
Solution 1
The first condition implies
So .
Putting each side to the power of :
so . Specifically,
so we have that
We only wish to find . To do this, we note that and now, by the Chinese Remainder Theorem, wish only to find . By Euler's Totient Theorem:
so
so we only need to find the inverse of . It is easy to realize that , so
Using Chinese Remainder Theorem, we get that , finishing the solution.
Solution 2 (Another way to find a)
Obviously letting will simplify a lot and to make the term simpler, let . Then,
Obviously, is times a power of . Testing, we see satisfy the equation so . Therefore, ~Ddk001
Alternate solution 1
If you've found but you don't know that much number theory.
Note , so what we can do is take and keep squaring it (mod 1000).
Alternate solution 2
Another way to find using modular arithmetic. In the same way as solution , we can find that. For some positive integers and . Taking the equation mod gives For some positive integer . Plug this back into the original equation.
~sdfgfjh
Video Solution by mop 2024
~r00tsOfUnity
See also
2017 AIME I (Problems • Answer Key • Resources) | ||
Preceded by Problem 13 |
Followed by Problem 15 | |
1 • 2 • 3 • 4 • 5 • 6 • 7 • 8 • 9 • 10 • 11 • 12 • 13 • 14 • 15 | ||
All AIME Problems and Solutions |
The problems on this page are copyrighted by the Mathematical Association of America's American Mathematics Competitions.